LSAT and Law School Admissions Forum

Get expert LSAT preparation and law school admissions advice from PowerScore Test Preparation.

User avatar
 Dave Killoran
PowerScore Staff
  • PowerScore Staff
  • Posts: 5852
  • Joined: Mar 25, 2011
|
#49521
Complete Question Explanation
(The complete setup for this game can be found here: lsat/viewtopic.php?t=17097)

The correct answer choice is (A)

The key to this question, and questions #4 and #5, is to Hurdle the UncertaintyTM. When X is added to class 1, the following diagram results:
S95_Game_#1_#3_diagram 1.png
The remaining variables are V, Z, W, Y, and T. Because the VZ pair and the WY pair cannot be added to the same class, they must occupy spaces in difference classes. But, space is limited in class 1 and class 3. Because no pair can be added to class 2, and in the reverse, no pair can be added to class 1 and class 3 (otherwise the remaining pair would have to both be added to class 2), we can deduce that one pair must be added to classes 1 and 2, and the other pair must be added to classes 2 and 3, as indicated by the straight lines in the diagram below:
S95_Game_#1_#3_diagram 2.png
This arrangement leaves only one remaining space, and that space must be filled by T:
S95_Game_#1_#3_diagram 3.png
Accordingly, answer choice (A) is correct.

Questions #3, #4, and #5 each trade on the same principle, but question #3 is considerably harder than questions #4 and #5. After you review the explanations to #4 and #5, return to this question and note how assigning the variable (X, in this case) to class 1 instead of class 3 changed the appearance of the problem. The underlying Hurdle the Uncertainty remains, but this problem is harder to solve because of that difference in variable assignment.
You do not have the required permissions to view the files attached to this post.
 Sara Gold
  • Posts: 12
  • Joined: Jul 11, 2015
|
#22244
Good afternoon,
I am reviewing my answers to the LSAT for September 1995 and was wondering how to best approach correcting some of the answers for the questions since no explanations are provided.

For some reason, I found the later Logic Games in this section easier, but had trouble with the first one.

For number 3, how do you quickly determine that T is the best option? I feel like it would take forever as you would have to hypothesize with each answer choice.

Thank you so much!
 David Boyle
PowerScore Staff
  • PowerScore Staff
  • Posts: 836
  • Joined: Jun 07, 2013
|
#22245
Hello Sara,

For question 3, you know that t in group 1 brings along z, which is too many folks. So it must be in group 2 or 3, and given the various hates (y hates w etc.), a lot of the ones left wouldn't fit well together if t is in group 3 (which fills up group 3 and prevents others escaping there), so it's group 2 for t.

Hope this helps,
David
 Sara Gold
  • Posts: 12
  • Joined: Jul 11, 2015
|
#22246
Thank you David, it does help!

Get the most out of your LSAT Prep Plus subscription.

Analyze and track your performance with our Testing and Analytics Package.